Difference between revisions of "2009 AMC 10B Problems/Problem 2"
m (→Solution) |
|||
(9 intermediate revisions by 6 users not shown) | |||
Line 15: | Line 15: | ||
</math> | </math> | ||
− | == Solution == | + | == Solution 1 == |
− | Multiplying the numerator and the | + | Multiplying the numerator and the denominator by the same value does not change the value of the fraction. |
We can multiply both by <math>12</math>, getting <math>\dfrac{4-3}{6-4} = \boxed{\dfrac 12}</math>. | We can multiply both by <math>12</math>, getting <math>\dfrac{4-3}{6-4} = \boxed{\dfrac 12}</math>. | ||
− | Alternately, we can directly compute that the numerator is <math>\dfrac 1{12}</math>, the denominator is <math>\dfrac 16</math>, and hence their ratio is <math>\dfrac 12</math>. | + | Alternately, we can directly compute that the numerator is <math>\dfrac 1{12}</math>, the denominator is <math>\dfrac 16</math>, and hence their ratio is <math>\boxed{(C)\frac{1}{2}}</math>. |
+ | |||
+ | == Solution 2 (Full Solution) == | ||
+ | We write both the numerator and denominator with a denominator of <math>12</math> first, since the LCM of <math>2,3,</math> and <math>4</math> is <math>3\cdot4=12</math>. | ||
+ | Next, we multiply both the numerator and denominator by <math>12</math>. <math>\dfrac{\frac{1}{3}-\frac{1}{4}}{\frac{1}{2}-\frac{1}{3}}=\dfrac{\frac{4}{12}-\frac{3}{12}}{\frac{6}{12}-\frac{4}{12}}=\dfrac{\frac{1}{12}}{\frac{2}{12}}=\boxed{(C)\dfrac{1}{2}}</math>. | ||
+ | -sosiaops | ||
== See Also == | == See Also == | ||
{{AMC10 box|year=2009|ab=B|num-b=1|num-a=3}} | {{AMC10 box|year=2009|ab=B|num-b=1|num-a=3}} | ||
+ | {{MAA Notice}} |
Latest revision as of 21:08, 27 December 2020
Problem
Which of the following is equal to ?
Solution 1
Multiplying the numerator and the denominator by the same value does not change the value of the fraction. We can multiply both by , getting .
Alternately, we can directly compute that the numerator is , the denominator is , and hence their ratio is .
Solution 2 (Full Solution)
We write both the numerator and denominator with a denominator of first, since the LCM of and is . Next, we multiply both the numerator and denominator by . . -sosiaops
See Also
2009 AMC 10B (Problems • Answer Key • Resources) | ||
Preceded by Problem 1 |
Followed by Problem 3 | |
1 • 2 • 3 • 4 • 5 • 6 • 7 • 8 • 9 • 10 • 11 • 12 • 13 • 14 • 15 • 16 • 17 • 18 • 19 • 20 • 21 • 22 • 23 • 24 • 25 | ||
All AMC 10 Problems and Solutions |
The problems on this page are copyrighted by the Mathematical Association of America's American Mathematics Competitions.